bull cat

August 25, 2017 | Author: yajur_nagi | Category: Poverty & Homelessness, Poverty, Obesity, Odds, Agriculture
Share Embed Donate


Short Description

bull cat 9 2015...

Description

2015 Bull Mock CAT - 09 DIRECTIONS for the question: Solve the following question and mark the best possible option. Question No. : 1 How many four-digit odd integers, greater than 3009 can be formed with the digits 0, 2, 3, 5 and 7, if repetition of digits is not allowed? A) 36

B) 32

C) 72

D) 54

Explanation:-   The unit’s digit can be filled in 3 ways (3, 5 or 7) and once this is done the thousand’s digit can be filled in 2 ways. The hundred’s and the ten’s digit can be filled up with remaining digits in 3 × 2 = 6 ways. Total number of ways = 3 × 2 × 6 = 36. Hence, total such four digit number greater than 3009 = 36 Question No. : 2 What is the digit at the hundredths place of the number N = 4536 ? A) 1

B) 0

C) 6

D) 5

Explanation:-   The last three digits of any number is the same as the remainder when the number is divided by 1000. 4536 / 1000 = 533 × 936 / 23 (Dividing numerator and denominator by 125) The remainder when 936 is divided by 8 will be 1 as  = (8 + 1)36 533 /8 = 5 × 2516 / 8 = 5 × (24 + 1)16/ 8 So the remainder when will be 533 is divided by 8 will be 5. So the remainder when 4536 is divided by 1000 will be 1 × 5 × 125 = 625 The required digit is 6.     Alternate explanation: We can write the term as (40 + 5)36 Now in the binomial expansion we get

4036 + 36 × 4035… +   36 ×  40 ×  535 + 536 Now, except for the last two terms, the rest of the terms will all have two or more zeroes at the end, so they will not contribute to the hundreds digit. If you look at the cycle of 5 powers, the last 3 digits are 125 for odd powers and 625 for even powers. So 536 will end in 625 Now 535 will end in 125. But when we multiply this with a 8, we end with 000. So even the second last term does not matter. Hence the hundreds digit will be 6.   Question No. : 3

A) 1/3

B) 1/6

Explanation:-  

C) 1/4

D) 1/5

Question No. : 4 Ankhush and Bimal started simultaneously from the same point and drove in opposite directions on a circular track of length 2520 m. initially Ankhush’s speed was 48 m/s and Bimal’s speed was 78m/s. Each time they met, they interchanged their speeds as well as the direction in which they traveled. Further, it is known that they met for the 46th time at 9:00pm   When did they meet for the 400th time? A) 2:18 am

B) 11:38 pm

C) 10:58 pm

D) 10:38 pm

Explanation:-  

The speeds are given (Ankhush, A = 48 m/s and Bimal, B = 78 m/s. Actually, A, B keep interchanging the speeds) The time interval between two A and B meet, each time they cover the total distance i.e., together they must cover the entire length of the track once to meet each other. Let them meet for the first time after t second In time t, A covers 48t and B covers 78t (in m) Now, 48t + 78t = 2520 => t = 2520/126 = 20 So, when they meet for the first time at X, A covers 48(20) = 960 m and B covers 78(20) = 1560 m. Next, A travels at 78 m/s in the clockwise direction and B travels at 48 m/s in the anticlockwise direction. To meet again, they need to together cover the total length of the track. So, after 20s, they meet at Y, A having covered 1560 m and B covering 960 m i.e., Y is 960 m from X in the anticlockwise direction. They meet each time they cover the entire length of the track, i.e., after every 20 seconds. Between their 46th meeting and 400th meeting the time gap is (400 – 46) (20) seconds = 7080 seconds = 118 minutes. Thus, they will meet for the 400th time 118 minutes from 9pm i.e., at 10:58 pm   Question No. : 5 Ankhush and Bimal started simultaneously from the same point and drove in opposite directions on a circular track of length 2520 m. initially Ankhush’s speed was 48 m/s and Bimal’s speed was 78m/s. Each time they met, they interchanged their speeds as well as the direction in which they traveled. Further, it is known that they met for the 46th time at 9:00pm   Which meeting will be their first at the starting point? A) 48th

B) 26th

Explanation:-  

C) 21st

D) 25th

The speeds are given (Ankhush, A = 48 m/s and Bimal, B = 78 m/s. Actually, A, B keep interchanging the speeds) The time interval between two A and B meet, each time they cover the total distance i.e., together they must cover the entire length of the track once to meet each other. Let them meet for the first time after t second In time t, A covers 48t and B covers 78t (in m) Now, 48t + 78t = 2520 => t = 2520/126 = 20 So, when they meet for the first time at X, A covers 48(20) = 960 m and B covers 78(20) = 1560 m.   Thus, they meet every time at a distance of 960 m from the previous meeting point in anticlockwise direction. To meet at the starting point 960 × n = 2520K. Where n is the required meeting when they meet at the starting point for the first time and K is the least natural number which satisfies the given condition.

So the minimum value of K = 8 (For n = 21). So they meet for the first time at the starting point when they are meeting for the 21st time.   Question No. : 6 A family decides to visit a temple on a hill top and begin their journey at 6a.m. The grandmother in the family is sick, and thus takes a Palki to reach the hill top. She leaves at 10a.m. so as to reach at the same time with the rest of the family. After reaching the hill top the family realizes that the grandmother hasn’t reached yet, gets worried and start walking down again. They meet the Palki at 1/3 the distance from the top. Afterwards, the Palki organizer tells them that the initially planned time was perfect but due to the sudden leave of a worker which occurred after travelling ¼ of the height, the speed of the Palki was reduced by 5/8 of its original speed. After what time did the family meet the grandmother again? A) 52 hours Explanation:-  

B) 69 hours 20 minutes

C) 48 hours

D) 75 hours 40 minutes

Question No. : 7 Which of the following is not a factor of 7396 – 4796 ? A) 24

B) 312

C) 65

D) None of these

Explanation:-   7396 – 4796 is divisible by (73 – 47) = 26 = 2 × 13 and (73 + 47) = 24 × 5 Also, 73 96 – 47 96 is divisible by 732 – 472

732 – 472 = 3120 = 2 × 5 × 312 Hence, 73 96 – 47 96 is divisible by 24, 65 and 312.   Question No. : 8 If a, b and c are real numbers such that a + b + c = 25 and ab + bc + ca = 75, what is the largest value that a can have? A) 65/3

B) 55/3

C) 25/3

D) None of these

Explanation:-     a + b + c = 25 ⇒c = 25 – (a + b) ab + bc + ca = 75 ⇒ ab + (b+a) c = 75 ⇒ab + (a+b) [25 - (a+b)] = 75 ⇒ab + 25a + 25b – a2 – b2 – 2ab = 75 ⇒b2 + (a - 25) b + (75 + a2 – 25a) = 0 As b is real

(a - 25)2 – 4(1) (a2 – 25a + 75) > 0 ⇒ a2 – 50a + 625 – 4a2 + 100a – 300 > 0 ⇒- 3a2 + 50a + 325 > 0

⇒ -3a2 + 65a – 15a + 325 > 0 ⇒(-3a + 65) (a+5) > 0 -3a + 65 < 0 and a + 5 < 0 ⇒ a > 65/3 and a 0 and a + 5 > 0 ⇒a < 65/3 and a > -5 The maximum value that a can have is 65/3. Question No. : 9 In a trapezium, when the mid-points of the diagonals are joined, the line so formed is 5 cm long. Further, if the length of one of the parallel sides of the trapezium is 25 cm, then find the length of the other parallel side. A) 10

B) 15

Explanation:-  

C) 20

D) 22.5

DIRECTIONS for the question: Go through the pie chart/s given below and answer the question that follows. Question No. : 10

What is the approximate average selling price of ice cream per ton (in millions Rs.) of all the six companies taken together? A) 13.3

B) 18.3

C) 23.3

D) 15.3

Explanation:-   Total sales of all the six companies taken together = Rs.64 billion or Rs.64000 million Average selling price (in million Rs.) per ton = 64000/3500 = Rs.18.3 million. Question No. : 11 The expenditure incurred in ice cream production by Dairy Queen was Rs.10.54 million per ton in the year 2012. What is the approximate percentage profit (as the percentage of expenditure) that Dairy Queen has earned? A) 27.2

B) 16.7

C) 44.4

D) 9.9

Explanation:-   Quantity of ice cream sold by Dairy Queen in the year 2012 = (15 × 3500)/100 = 525 tons Total expenditure incurred (in million Rs.) by Dairy Queen = 10.54 × 525 = 5533.5 Total sales achieved by Dairy Queen = (11 × 64000)/100 = Rs. 7040 million Dairy Queen‘s profits in the year 2012 = 7040 – 5533.5 = Rs. 1504.5 million  So, approximate profit percentage = (1504.5 × 100) / 5533.5 = 27.2 %

  Question No. : 12 What is the approximate marked price (in million Rs.) per ton of Dippin Dots, if the discount per ton of Dippin Dots is 10% of the selling price? A) 22.4

B) 15.7

C) 27.8

D) 35.5

Explanation:-   Marked price = Selling price + Discount    Quantity of the ice cream sold by Dippin Dots = (21 × 3500)/100 = 735 tons Total sales of Dippin Dots = (29 × 64000)/100 = Rs.18560 million Now, per ton selling price of Dippin Dots' ice cream = 18560 / 735 = Rs. 25.25 million  Discount = 10% of 25.25 = Rs. 2.525 million So, marked price = 25.25 + 2.525 = Rs. 27.775 million or Rs.27.8 (approx.).   Question No. : 13 It is known that the profits earned by Blue Bunny and Dione are 20% and 25% of their respective total sales of the year 2012. What is the approximate ratio of the cost price per ton of Blue Bunny's ice cream to that of Dione ice cream? A) 5 : 8

B) 5 : 2

C) 9 : 17

D) 14 : 11

Explanation:-   Quantity of the ice cream sold by Blue Bunny = (18 × 3500)/100 = 630 tons Total sales of Blue Bunny = (17 × 64000)/100 = Rs. 10880 million Now, per ton selling price of Blue Bunny's ice cream = 10880 / 630 = Rs. 17.27 million  So, cost price per ton of Blue Bunny's ice cream = 0.8 × 17.27 = Rs. 13.82 million   Quantity of ice cream sold by Dione in the year 2050 = (16 × 3500)/100 = 560 tons Total sales of Dione = (13 × 64000)/100 = Rs. 8320 million Now, per ton selling price of Dione's ice cream = 8320 / 560 = Rs. 14.86 million  So, cost price per ton of Dione's ice cream = 0.75 × 14.86 = Rs. 11.15 million   Therefore, the required approximate ratio = 14/11  

DIRECTIONS for the question: Solve the following question and mark the best possible option. Question No. : 14 The figure below is composed of a collection of square arranged to form a rectangle the squares are numbered 1 to 10. If the dimensions of the squares are all distinct integers then the dimensions of the smallest rectangle that can be formed by these squares are

A) 57 × 60

B) 104 × 105

C) 156 × 160

D) 208 × 210 

Explanation:-  

  The dimensions of this smallest rectangle are 104 by 105. Question No. : 15

In the above diagram what is the difference between the areas of 5th square and 7th square? A) 600 units

B) 548 units

C) 128 units

D) None of these

Explanation:-  

 

The difference in the area = 282 – 162 = 528. DIRECTIONS for the question: Solve the following question and mark the best possible option. Question No. : 16 Ajay, Vipin and Anil started from the same place and travelled in the same direction at the speeds of 80, 70 and 60 kmph respectively. Vipin started 3 hours after Anil and both Ajay and Vipin overtook Anil at 9.00 p.m. of the same day. Find the time at which Ajay started? A) 5:15 a.m.

B) 5:30 a.m.

C) 6:00 a.m.

D) 5:45 a.m.

Explanation:-   Distance covered by Anil in the 1st 3 hours = 180 km

Time taken by Vipin to overtake Anil = 180 / (70 – 60) = 18 hours Distance traveled by Vipin in this time = 18 × 70 = 1260 km Given that the distance traveled by Ajay in this time is also 1260 km. Therefore, time in which Ajay overtook Anil = 1260/80 = 15.75 hours Now subtracting 15.75 hours from 9 p.m. the time happens to be 5:15 a.m. Thus Ajay started at that time.   Question No. : 17 The selling prices of three Brands of sugar X, Y, and Z are in the ratio 1 : 4 : 2  and their cost prices are in the ratio 3 : 7 : 5 respectively.             What is the cost price of Brand Z? I. The difference between the cost price and selling price of all the three Brands is the same and is equal to Rs.200 II. Profit is made on only one of the three Brands. A) If the question can be answered by any one of the two statements alone but not by the other statement alone B) If the question can be answered by either of the two statements alone C) If the question can be answered only if both the statements are taken together D) If the question can be answered only if both the statements are taken together. Explanation:-   Let the cost price (c.p) and selling price (s.p) of X, Y, Z is represented as follows.  

X

Y

Z

c.p

3x

7x

5x

s.p

y

4y

2y

Difference

|3x – y|

|7x – 4y|

|5x – 2y|

From statement I: Given all the difference a equal i.e. |3x – y| = |7x – 4y| = |5x – 2y| case 1: if all the Brands are sold at either profit or loss, then 3x – y = 7x – 4y = 5x – 2y 3x – y = 7x – 4y and 7x – 4y = 5x - 2y Data insufficient Case 1: is invalid Case 2: If the loss is made only on Z or loss is made only on X and Y. Then 3x – y = 7x – 4y = 2y – 5x 3x – y = 7x – 4y and 7x – 4y and 2y – 5x

Data is inconsistent Case 2 is invalid Case 3: if loss is made only on Y or loss is made only on X and Z then

Case 3 is valid Case 4: if loss is made only on X or loss is made only on Y and Z then y – 3x = 7x – 4y = 5x – 2y y – 3x = 7x – 4y and 7x – 4y = 5x – 2y 5y = 10x and 2x = 2y

Data inconsistent Case 4 is invalid

∴ loss is made only on Y or loss is made only on X and Z Difference of C.P and S.P of X = |3x – y| x = K, y = 2K ⇒d = |3K – 2K| = K ∴Loss is made on X and Z Given difference = 200 => K = 200 Cost price of Brand Z = 5x = 5 × 200 = 1000 ∴statement I alone is sufficient From statement II alone, we cannot infer anything   Question No. : 18 The probability of a bomb hitting its target is ½ and it takes 3 bombs to destroy a building. What is the minimum number of bombs to be fired so that the probability of the building being destroyed exceeds 0.95? A) 9

B) 10

C) 11

D) 12

Explanation:-  

Question No. : 19 If f(x) + f(1 – x) + f(1 + x) + f(2 + x) = 2x for all real values of x, and f(0) = 1, then find the value of f(4)?   A) 4

B) 5

C) 6

D) 7

Explanation:-   For x = 0, f(0) + f(1) + f(1) + f(2) =0                                                 (a) For x = 1, f(1) + f(0) + f(2) + f(3) = 2                                                (b) Subtract (a) from (b), we get f(3) – f(1) = 2                                                              (A) For x = 2, f(2) + f(-1) + f(3) + f(4) = 4                                              (c) For x = - 1, f(-1) + f(2) + f(0) + f(1) = -2                                           (d) Subtract (c) from (d), we get f(0) + f(1) – f(3) – f(4) = - 6 f(0) – f(4) – 2 = -6 [from (A), we know that f(3) – f(1) = 2] Also its given, f(0) = 1, so we get f(4) = 5.     DIRECTIONS for the question: Solve the following question and mark the best possible option.

Question No. : 20 If a, b, c, d are different integers such that (x – a)(x – b)(x – c)(x – d) = 4. How many integral values (n) of x are possible? A) 0 ≤ n ≤ 2

B) 3 ≤ n ≤ 7

C) 8 ≤ n ≤ 15

D) None of these

Explanation:-   Infinite values of x are possible. Product of different integers is 4. Integers can be 2, 1, -1 and -2. Consider one case where x = 0, a = -2, b = -1, c = 1, d = 2. Similarly x = 1, a = -1,b = 0,c = 2,d = 3 and x = 2, a = 0, b = 1, c = 3, d = 4 also satisfy the equation. We can go on increasing the value of each variable by one to get infinite solutions.  

DIRECTIONS for the question: Go through the pie chart/s given below and answer the question that follows. Question No. : 21 Figure 1

 

Figure 2

What is the total number of voters of INC in Maharashatra if the total number of voters of CPI in India is 47,022 and total voters who vote in Maharashtra is 1,530,200? A) If the question can be answered by figure 1 alone and figure 2 is not required. B) If the question can be answered by figure 2 alone and figure 1 is not required. C) If the question can be answered by both of the figures taken together and not by any one of these figures. D) If the question cannot be answered even by using both the figures together.

D) If the question cannot be answered even by using both the figures together. Explanation:-   Using Figure 1 alone, the total number of voters of CPI = 47022 So total voters in India = (47022 × 100)/ 3 = 1,567,400 Total voters of INC = (34 × 1567400)/100 = 532,916 Total number of voters in Maharashatra = 1,530,200 But Figure 1does not provide any relation between Maharashatra voters and INC voters So Figure 1 is not sufficient.   Using Figure 2 alone, we know the total number of CPI = 47022 and the total number of voters of Maharashatra = 1,530,200.  But we don’t know about the total number of voters of INC.   So, using Figure 2 alone is also not sufficient.   Using both figures 1 & 2, Total no. of voters of INC = 532,916 Percentage distribution in Maharashatra = 28 So, the total number of voters of INC in Maharashatra = (28 × 532916)/100 = 149216.48 or 149,216 So using both figures the question can be answered.  

DIRECTION for the question: Refer to the data below and answer the question that follow.  Apart from the usual racing scene at the Kharga Race course where huge bets are placed and high stakes are involved, the Race Course Committee organizes a Mela every year during winter. This is a unique concept basically aimed at publicity of the Race Course, and popularizing racing among people. On Sunday every week a certain fixed number of races take place. A maximum of 1428 spectators can be accommodated on a day. The ticket is worth Rs. 500/- of which 20% is retained by the management and 20% is retained for paying horse racers. Each spectator is allowed to speculate the winner of anyone of the scheduled races. When all the races are over, the rest of the money collected is divided equally among those who have speculated the winners correctly. In Jan 2009, a spectator took home Rs. 8,270 in the first week. The next week, another took home Rs. 12,405, the same number of spectators being present as the first week. Question No. : 22 How much money was divided among the spectators who speculated correctly in the first week? A) Rs. 6,48,000

B) Rs. 4,55,000

C) Rs. 2,48,100

D) Cannot be determined

Explanation:-  

Hence, option 3.

Question No. : 23 What was the amount retained by the management in the first week?

A) Rs. 82,700

B) Rs. 100,000

C) Rs. 116,250

D) Cannot be determined

Explanation:-  

  Hence the amount retained by the management is Rs. 82700. Option A.

DIRECTIONS for the question: Solve the following question and mark the best possible option. Question No. : 24 In ΔABC, we have ∠A = 1000 and ∠B = ∠C = 400. The side AB is produced to a point D so that B lies between A and D and AD = BC. Then ∠BCD is A) 100

B) 120

C) 150

D) 300 

Explanation:-  

DIRECTIONS for the question: Read the information given below and answer the question that follows. Question No. : 25 The Indian cricket team had just finished a five-match series with Pakistan and the team management was assessing the performance of the top five batsmen: Dravid, Dhoni, Sehwag, Sachin and Kaif - who played and got out in each of the five matches. For each of the five matches, the table below gives the names of the three highest scoring batsmen in that match, along with their respective scores, mentioned in the brackets alongside. The last column gives the percentage of the total team score made by the given three players together. For the question, assume that total team score is made by these five players only. The team management has devised the following two indices to judge the performance of each player:   1. The Consistency Index (C.I.), which is the middle number, if all the five scores of the concerned player in the series are arranged in the increasing order, 2. The Reliability Index (R.I.), which is the difference between the highest and the lowest scores of the concerned player in the series.   Match

Highest

Second highest

Third highest

Percentage of total score

1

Dravid (97)

Dhoni (63)

Sehwag (56)

80

2

Sachin (85)

Kaif(65)

Dravid (30)

75

3

Dhoni (125)

Kaif(62)

Sachin (37)

70

4

Sachin (77)

Dravid (62)

Kaif (48)

85

5

Sehwag (80)

Dhoni(72)

Dravid (60)

80

  Further, it is also observed that in any match, the scores of no two players who batted are equal.   Dhoni and Sehwag together accounted for at most what percentage of India's total score in the series, approximately? A) 42%

B) 38%

C) 40%

D) 44%

Explanation:-   India’s total score in the five matches are 270, 240, 320, 220 and 265 respectively. The scores of the five players (the possible range in case the exact values cannot be found) in the matches are   Match Dravid Dhoni Sehwag  Sachin Kaif 1

97

63

56

0-54

054

2

30

0-29

0-29

85

65

3

0-36

125

0-36

37

62

4

62

0-33

0-33

77

48

5

60

72

80

0-53

053

 

Question No. : 26 Among the players mentioned, who can have the lowest value of R.I in the series? A) Only Dravid, Sehwag, Sachin or Kaif

B) Only Kaif

C) Only Kaif or Sachin

D) Only Kaif, Sehwag or Sachin

Explanation:-   The range of R.I. for the different players are 1.

Dravid

67-97

2.

Dhoni

96-125

3.

Sehwag

51-80

4.

Sachin

48-85

5.

Kaif

17-65

The value of R.I. can be minimum for Sehwag or Sachin or Kaif.  

DIRECTIONS for the question: Solve the following question and mark the best possible option.

Question No. : 27 The pendulum of a clock takes 7 seconds to strike 4 o'clock. How much time (in seconds) will it take to strike 11 o'clock? A) 11/7

B) 7

C) 70/3

D) 77/4

Explanation:-   The more relevant thing here is not the number of strikes but the number of time intervals. In other words, if a clock has to strike 4, there are 3 time intervals between the 4 strikes (the 1st strike happens at the 0th sec). So, in 7 seconds, the pendulum had 3 time intervals. To strike 11, there have to be 10 time intervals which will take (10 × 7)/3 = 70/3 seconds.  

Question No. : 28 What is the number of integral values P can take, where 2 ≤ P ≤ 30, such that the product (P – 1) × (P – 2) ….. (3) × (2) × (1) is not divisible by P? A) 9

B) 10

C) 11

D) 8

Explanation:-   (P – 1)! is not divisible by P when P is Prime. There are 10 prime numbers in this range: 2, 3, 5, 7, 11, 13, 17, 19, 23, 29. Also P = 4 satisfies this condition hence total numbers are 10 + 1 = 11. Question No. : 29 The odds against a certain event P are 7 : 4 and the odds in favour of another independent event Q are 8 : 7. The probability that at least one of the events will happen is: A) 11/165

B) 59/165

C) 116/165

D) 118/165

Explanation:-   The probability of P occurring = 4 / (7 + 4) = 4/11 Odds in favour of Q = 8 : 7 Probability of Q occurring = 8 / (8 + 7) = 8/15   Therefore, the probability that at least one of the events will happen = 1 – (7/11 × 7/15) = 1 –  49/165 = 116/165   Question No. : 30 How many spectators were present for the second week’s racing? A) 937

B) 827

Explanation:-  

C) 763

D) Cannot be determined

Hence, option 2. Question No. : 31   Debarati assigned the first 11 natural numbers as a, b, c, d, e, … , k not necessarily in that order. After some days, she forgot which number corresponds to which letter, but she knew that a + b + c + d + e + g + h + i + k = 3f + 5j. Which of the following number cannot be the value of j? A) 9

B) 5

C) 7

D) 3

Explanation:-  

Question No. : 32 7,6,5,4,3,2,1 are seven friends - start driving the car from the common point having speed in the ratio 1: 2 : 3 : 4 : 5 : 6 : 7 respectively, and start racing around a circular track. If 7, 5, 3, and 1 run in the same direction, while the remaining run in the opposite direction, at how many distinct points on the track does 7 meet any other runner? A) 17

B) 18

Explanation:-  

C) 12

D) None of these

Question No. : 33 Let a, b, c be the respective numbers that show up when three fair dice, P1, P2 and P3, are rolled. If z = |(a – 1) (b – 3) (c – 4)|, the probability that z 1 is   A) 

B) 

C) 

D) 

Explanation:-  

Question No. : 34 A basket contains 15 apples, exactly 6 of which are rotten. Each day one apple is taken out from the box. What is the probability that after four days there are exactly 8 apples in the basket that are not rotten? A) 

B) 

C) 

D) 

Explanation:-  

Question No. : 35

A) 51

B) 55

C) 54

D) 52

Explanation:-  

Question No. : 36 If all the factors of the number 129600 which are divisible by 5 are multiplied then the product would be A) 2210 × 3102 × 5140 Explanation:-  

B) 2210 × 3140 × 5105

C) 2140 × 3210 × 5102

D) 2140 × 3102 × 5210

Question No. : 37

A) 2cm

B) 2.5cm

C) 2.25cm

D) 3cm

Explanation:-  

Question No. : 38 A triangle has sides measuring 10, 17 and 21 units.A square is inscribed in the triangle such that one side of the square lies on the longest side of the triangle. The other two vertices of the square  touch the two shorter sides of the triangle. What is the length (in units) of the side of the square?

A) 

B) 

C) 

D) 

Explanation:-  

Question No. : 39 Consider a function g(n) = n3 – n2 (3 + t) + n(2 + 3t) – 2t, such that t is a prime number other than 2. What is the range of values of n that satisfy g(n) > 0? A) n < 1 or 2 < n t

C) 1 < n < 2  or n > t

D) n < 1 or n > t

Explanation:-  

Given   g(n) = n3 – n2 (3 + t) + n( 2 + 3t) – 2t > 0 ⇒ n3 – n2t + 2n – 2t + 3tn  – 3n2 > 0

⇒ n2(n – t) + 2(n – t) – 3n(n – t) > 0 ⇒ (n2 – 3n + 2)(n – t) > 0 ⇒ (n – 1)(n – 2)(n – t) > 0 Since, t is an odd prime number, therefore t is not less than 3. Therefore, the range of values of n for which g(n) > 0 is  1 < n < 2   or   n > t.   Question No. : 40 Ria has three types of boxes viz. red, blue and green. She plays a game in which she placed 9 red boxes on the table. She puts 5 blue boxes each, in a few of the red boxes then she puts 5 green boxes each, in few of the blue boxes. If the number of boxes that have been left empty in the game is 41, then how many boxes were used in the game by Ria? A) 72

B) 49

C) 63

D) 56

Explanation:-   Let the total number of red boxes that have been left empty in the game = x Therefore, the total number of blue boxes used by Ria

is 5 × (9 – x) = 45 – 5x Let the total number of blue boxes that have been left empty in the game = y Therefore, the total number of green boxes used by Ria is 5 × (45 – 5x – y) = 225 – 25x – 5y It is also known that total number of boxes that have been left empty = 41 Therefore, x + y + 225 – 25x – 5y = 41 24x + 4y = 184.  6x + y = 46 Total number of boxes used by Ria in the game is 9 + 45 – 5x + 225 – 25x – 5y = 279 – 5(6x + y)  = 279 – 5 × 46  = 49 Question No. : 41 Three friends are playing a card game. They start with sums of money in the ratio 7 : 6 : 5 and finish with sums of money in the ratio 6 : 5 : 4, in the same order as before. One of them won Rs. 12. How many rupees did he start with? [The three friends gambled amongst each other only] A) Rs. 1080

B) Rs. 420

C) Rs. 210

D) Rs. 108

Explanation:-  

Question No. : 42

A) 6

B) 

C) 

D) 

Explanation:-  

Question No. : 43 The labor cost of making a bicycle is Rs. 1,347. Find the new cost of making the bicycle if the hourly wage rate of the workers is increased by 12.5 % and the working hours per day increased by 5 %? A) Rs. 1243

B) Rs. 1592

Explanation:-  

C) Rs. 1443

D) Rs. 1321

Question No. : 44 Atul has exactly six sealed bags containing 15, 31, 19, 20, 16 and 18 candies. Out of the six bags with Atul, there are exactly five bags that contained chocolate candies whereas one box contained orange candies. He distributed all the six bags among his three sons in such a manner that his eldest son got the only box with orange candies and the other bags were distributed in such a manner so that other two brothers received the chocolate candies in the ratio of 2 : 1. How many orange candies were there with Atul? (Assume no candies were taken out of the bags) A) 20

B) 19

C) 16

D) 31

Explanation:-   The total number of chocolate candies has to be multiple of three i.e, the total number of candies in five out of the six bags should be a multiple of three. 15 and 18 are already in the form 3n.  We see that three bags (containing 31, 19 and 16) have (3n + 1) number of candies each, and one box containing 20 candies is of the form (3n + 2). Hence, the total number of orange candies has to be 20 as the candies in the other five bags sum up to a multiple of three. DIRECTIONS for the question: The question below is followed by two statements marked A and B. Mark as your answer. Question No. : 45 A barman has 25 small casks, all of which are completely filled with wine. He also has one large cask which is empty. Can he transfer all the wine contained in 15 of these small casks to the large cask?   A.  The large cask has a capacity of 500 liters. B.  The average capacity of any 20 of the 25 small casks is 25 liters.   A) If the question can be answered with the help of the information contained in either statement alone; B) If the question can be answered with the help of the information contained in both statements together; C) If the question can be answered with the help of information contained in any one statement alone; but not by the other statement alone; D) If the question cannot be answered even with the help of information contained in both statements together Explanation:-   Using statement A: We get the volume of the large cask but we have no information about the total capacity of the 15 small casks. Hence, statement A is not sufficient. Using statement B: We have no information about the volume of the large cask. But we can conclude that every cask must have the same volume of 25 litres, since any 20 casks has the average as 25 litres. Combining both the statements together: Since the volume of the large cask is 500 litres and the volume of each of the small cask is 25 litres, therefore the wine contained in 15 such small casks can be transfered into the large cask. Question No. : 46 A wire of length l units is cut into three pieces having lengths a units, b units and c units. If a > b > c and each of a, b and c is an integer, then is b an odd number?   A.  The product of a and b is 60 square units. B.  The product of b and c is 12 square units.   A) If the question can be answered with the help of the information contained in either statement alone; B) If the question can be answered with the help of the information contained in both statements together; C) If the question can be answered with the help of information contained in any one statement alone; but not by the other   statement alone; D) If the question cannot be answered even with the help of information contained in both statements together

Explanation:-   Using statement A: Given that a × b = 60. The values of b and a can be (2, 30), (3, 20), (4,15), (5,12), (6,10). Therefore, b can be either odd or even. Hence, statement (A) alone is not sufficient to answer the question. Using statement B: Given that b × c = 12. The values of c and b can be (1, 12),  (2, 6) and (3, 4) in that particular order as and c < b So, in each of the cases, b is an even number. Hence, statement (B) alone is sufficient to answer the question. Question No. : 47 Sales value of a company ‘X’ is Rs. 100 lacs and the margin is 20% in the year 2000. What is the total cost to the company in the year 2001?   A. Sales value in the year 2002 is 80% of the sales value in the year 2001 and 125% of the sales value in the year 2003. Profit percentage is constant from the year 2000 to the year 2003. B. Sales value in the year 2003 is 140% of the sales value in the year 2000.   A) If the question can be answered with the help of the information contained in either statement alone; B) If the question can be answered with the help of the information contained in both statements together; C) If the question can be answered with the help of information contained in any one statement alone; but not by the other statement alone; D) If the question cannot be answered even with the help of information contained in both statements together Explanation:-  

DIRECTIONS for the question: Study the table/s given below and answer the question that follows. Question No. : 48

Find the number of female students who obtained more total marks than at most four female students and more total marks than at least one male student. A) 1

B) 2

C) 3

D) 4

Explanation:-   The total marks obtained by the students and their overall ranks are tabulated in the following table:   NAME

Gender College Rank Total

Dev

M

I

7

40

Priya

F

II

12

36

Anup

M

III

9

38

Pranay

M

I

3

50

Ashok

M

III

2

52

Sheela

F

II

11

36

Reena

F

I

16

27

Rohan

M

II

1

53

Sarika

F

I

6

41

Mani

M

II

5

44

Nidhi

F

III

15

33

Ariti

F

III

10

38

Agni

F

II

4

46

Atul

M

I

8

39

Salil

M

II

13

34

Niti

F

III

14

33

  Two female students namely Priya and Sheela have obtained more marks than one male student namely Salil and more marks than three female students namely Reena, Nidhi and Niti. Question No. : 49 If the criterion for ranking the students is followed by each center to rank the students enrolled there, then find the difference between the ‘overall rank’ and the ‘center rank’ of Atul. (Overall rank is the rank when all the 16 students are taken into consideration and ‘center rank’ is the rank when only the students of that particular center is taken into consideration.) A) 6

B) 2

C) 5

D) 4

Explanation:-   The total marks obtained by the students and their overall ranks are tabulated in the following table:   NAME

Gender College Rank Total

Dev

M

I

7

40

Priya

F

II

12

36

Anup

M

III

9

38

Pranay

M

I

3

50

Ashok

M

III

2

52

Sheela

F

II

11

36

Reena

F

I

16

27

Rohan

M

II

1

53

Sarika

F

I

6

41

Mani

M

II

5

44

Nidhi

F

III

15

33

Ariti

F

III

10

38

Agni

F

II

4

46

Atul

M

I

8

39

Salil

M

II

13

34

Niti

F

III

14

33

  Overall rank of Atul is 8 and center rank of Atul is 4. Therefore, the required difference is 8 – 4 = 4. Question No. : 50 From which center, the maximum possible number of students obtained a total of at least 40 marks and at most a total of 54 marks? A) Both I and II

B) III

C) II

D) Both I and III

Explanation:-   The total marks obtained by the students and their overall ranks are tabulated in the following table:   NAME

Gender College Rank Total

Dev

M

I

7

40

Priya

F

II

12

36

Anup

M

III

9

38

Pranay

M

I

3

50

Ashok

M

III

2

52

Sheela

F

II

11

36

Reena

F

I

16

27

Rohan

M

II

1

53

Sarika

F

I

6

41

Mani

M

II

5

44

Nidhi

F

III

15

33

Ariti

F

III

10

38

Agni

F

II

4

46

Atul

M

I

8

39

Salil

M

II

13

34

Niti

F

III

14

33

  From college I as well as college II, there are three students who have obtained at least 40 and at most 54 marks. DIRECTIONS for the question: Identify the most appropriate summary for the paragraph. Question No. : 51 Virtually no job comes without stress. Whether it’s meeting the expectations and deadlines of coworkers, clients, or supervisors, nearly all work can at times be challenging. Sometimes the work itself isn’t as challenging as managing relationships with the people we work with.  Emotional labor involves managing our emotions to meet our job expectations.  For example, retail clerks are expected to be upbeat and enthusiastic about the merchandise (and in general), even if that is not truly how they feel. Emotional labor is also part of dealing with the personalities of those we work with. This labor is not necessarily always stressful. Asking a coworker about a sick relative may be a way to convey your concern about their family without taking much of an emotional toll. But in other cases emotional labor can be very stressful, and this stress can be minimized or magnified based on one’s status. A) Emotional labor is the primary contributor of stress at workplaces and it needs to be minimized for one to be productive. B) Emotional labor is far major contributor of stress at workplaces than the work itself and it needs to carefully understood in order to improve working conditions.  C) Emotional labor is a contributory factor to the stress experienced at workplaces and it needs to be understood in order to establish its impact. D) Emotional labor plays a role in the stress experienced at workplaces and it has an impact on a person’s job efficacy. Explanation:-   Option D This is a tricky question where you need to establish the subtle differences between the various answer options. Options 1 and 2 are rejected as they label emotional labor as the primary/major contributor of stress; this is a sentiment that is missing from the given paragraph. Options 3 and 4 are identical for the first part of answer option and only the second part varies. Option 3 states that emotional labor needs to be understood in order to establish its impact. This is something that is not mentioned in the paragraph. On the other hand, the second part of option 4 simply means that emotional labor has an impact on a person’s performance, a fact that can be easily deduced from the given context. Question No. : 52 Affective responses to material objects are integral to their biographical meaning to their owners and their participation in intimate relationships. Writers on material culture and affect have noted the entangling of bodies/selves with physical objects and how artefacts act as extensions or prostheses of the body/self, becoming markers of personhood. Objects become invested with sentimental value by virtue of their association with specific people and places, and thus move from anonymous, mass-produced

items to biographically-inscribed artefacts that bear with them personal meanings. Over use and with time, such initially anonymised objects become personalised prosthetics of the self, their purely functional status and monetary value replaced by more personal and sentimental value. A) In the material world, over a period of time, most objects obtain a certain sense of personlisation and attain a certain position in the life of their owner. B) Material objects, though devoid of life and ability to emote, paradoxically hold the power to associate with their owner and develop an attachment that has a personal and sentimental value. C) With material objects, there is a chance that the owner might see them more than just bulk manufactured items and might attach a sentimental or personal value of these items. D)Material objects, through a process of association, move away from just being factory-produced items to things that hold   emotional value for their owner.  Explanation:-   Option D You need to be really careful with the option you select for this question. Most of the answer options in this question are very close and carefully consider each before selecting the right answer. Remember, this is a paragraph summary question and you need to identify the answer that carries the most significant points of the paragraph. Let us analyze each option one at a time: Option 1: The option uses the phrasing ‘most objects’, something which is not stated in the passage. Option 2: This option almost treats a material object as something that has some conscious power to bring about a certain effect; the option assigns power to these objects which is actually incorrect. The paragraph states that the owners attach themselves to objects and not the other way round. Option 3: The paragraph does not talk in terms of ‘chances’ and is simply describing a certain phenomenon that takes place with material objects when their owners assign value to them. Option 4: This is the apt answer as it covers the central aspect of the passage and does not commit any mistake (like the options above).    DIRECTIONS for the question: Read the passage and answer the question based on it. Question No. : 53 A recent study in the Lancet medical journal concluded that in 2008, about 146 million adults globally were overweight and 502 million were obese. It may seem strange to be worried about too much food when the United Nations suggests that, as the planet’s population continues to expand, about 1 billion people may still be undernourished. The issue isn’t so much that we can’t grow enough. Rather, existing food supplies are so poorly distributed that those hundreds of millions have too little for their own health, while 2 billion-plus have too much. Even within families, malnutrition is often a distribution issue: How else to explain that about one in 10 households in Russia contain both underweight and overweight members? We’ve known the solution to starvation is usually very simple: Ensure poor people have enough money to buy food.   As poverty declines—and the percentage of the population worldwide living on less than $1.25 a day has halved since 1990— fewer people will be too poor to buy enough to eat healthily. The Lancet study reports that the relationship between income and nutritional status breaks down after countries reach an average income of $5,000.   When it comes to food, we are living in a world of plenty. For those worried about agricultural sustainability, there is a lot of slack in the system. A third of food production is simply wasted worldwide—spoiled before it reaches consumers or thrown away after that. Continued increases in agricultural productivity, thanks to new seed varieties and more efficient farming practices like fertilizer micro-dosing and drip irrigation, mean that sustainably feeding the world’s population, even if it grows past 9 billion, is eminently achievable.   The bad news is that the global obesity epidemic is a more complex problem than the conditions that felled most poor people in the past. Many common killers like measles can be prevented by a vaccine, malaria can be battled with bed nets and insecticide spraying, and diarrhea is a condition where large quantities of sugar water is actually a plus—add a little salt and you’ve got the perfect treatment for dehydration.   Obesity, on the other hand, has a whole range of different causes and no simple public health solution. The increasing numbers of people worldwide who earn a living sitting down rather than moving around, as services overtake agriculture as the biggest employer, mean the amount of calories the average human needs to consume is actually falling. But agricultural productivity has

led to a dramatic long-term decline in the cost of food at a time when growing wealth is providing more resources to buy sugary and fatty products.    Excerpted from an article in Business Week.   In the paragraph that follows the passage, we could expect the author to discuss A) Statistics related to Obesity in different countries C) Proposed solutions to the problem of obesity.

B) Description of different types of obesity. D) Diseases which result from obesity.

Explanation:-   Option 3. The passage talks about Obesity and many common killers like measles , malaria, and diarrhea. The author states that it is easier to tackle the common killers because their cause can be identified. However obesity has a whole range of different causes. Hence what would logically follow would be the solution to this problem of Obesity.

  1. Statistics about obesity have already been mentioned in the first paragraph.   2, 4. The author has just finished talking about the complexity of solutions to obesity - so it would not be in place to then discuss types of obesity or diseases which result from obesity. If at all it was required, it would have been done in the second para. Question No. : 54 Of the following, which represents a public health solution to the problem posed in the passage? A) Having a good Public Distribution System in place to prevent malnutrition deaths. B) Force life style change by ensuring a 2 year compulsory service in the armed forces for all youth. C) Reduction in farm wastage of food by having an efficient cold chain. D) Education about life-style changes through individual counseling. Explanation:-   Option D. Refer to the lines: Obesity, on the other hand, has a whole range of different causes and no simple public health solution. The increasing numbers of people worldwide who earn a living sitting down rather than moving around, as services overtake agriculture as the biggest employer, mean the amount of calories the average human needs to consume is actually falling. But agricultural productivity has led to a dramatic long-term decline in the cost of food at a time when growing wealth is providing more resources to buy sugary and fatty products. In the given question, we need to address the problem from a public health perspective and from a viewpoint that strikes as the core of the issue. Two answers fit the backdrop of the question: option B and option D. The difference between these two options is that that option B talks about a forced one and is not a solution for the general populace. Option D approaches the question from a general perspective and resolves the issue. Question No. : 55 With reference to the second paragraph, what could be the reason for the lack of correlation between income and nutritional status after the 5,000 dollar mark? A) Once a country is over that line, the considerable majority of people have the ability to eat enough, and the choice to eat   too much. B) Spending on food as a percentage of the total expenditure reduces after this mark, as families start spending more under other heads. C) Service sector starts dominating after this mark, which often signifies jobs that involve very little physical activity. D) As income rises, consumption of fatty foods, particularly non-vegeterian foods, increases disproportionately. Explanation:-   Option A. The context here is a correlation between nutritional status and income. At very low levels, people do not have enough to buy basic food - so their nutritional status is low. It keeps on increasing with income, but at one point when the physiological need of

nutrition has been done, then the correlation breaks down. Though old habits die hard, so people probably keep on increasing intake of food, even when it is not required.   2 is right - but it does not offer an explanation to the non-correlation.   3 may be true, but it only signifies a very tenuous relationship with nutrition - which the question is asking you to discuss.   4 is also true, but it can be viewed as a subset of the increased eating which is mentioned in option 1.   DIRECTIONS for the question: Given below are sentences that form a paragraph, identify the sentence(s) or part(s) that is/are incorrect in terms of grammar and usage (including spelling, punctuation and logical consistency). Then, choose the most appropriate option. Question No. : 56 A.  Both in the media and elsewhere there is a backlash against women’s participation as elected representatives in the panchayat system. B.  The allegation of “ proxy” rule follows from the perception that it is the men who are ruling through their wives which are mere mouthpieces. C.  It is true that women do get advice and help from her husband. D.  But does this in itself constitute “proxy” rule? What about the kitchen cabinets of powerful politicians? E.   What about the close kinship and other networks that leaders here and abroad draw upon for support?   A) B and E only

B) C  only

C) B and C only

D) A and E only

Explanation:-   Option 3. In B, which to be replaced by who as we are talking of people – wives. In C, their husbands and not her husband.  

DIRECTIONS for the question: The question consists of five statements labelled A, B, C, D and E which when logically ordered form a coherent passage. Choose the option that represents the most logical order. Question No. : 57 A.    Insects do much of what people do: they meet, mate, fight, and part with what resembles love or animosity. B.    Beetles care for their young and wasps engage in horrific battles. C.    Yet they do these things in stunningly different ways from humans, accomplishing similar goals without any of the same means, lacking vertebrates’ large brains and a complex system of hormonal signals. D.   That insects don’t need a big brain to do big things forces us to think harder about what is required to evolve complex behavior. E.    Going beyond the simplistic view of insect life in human terms we can’t take the easy way out and assume that a dragonfly is jealous of a rival or that a mother earwig is sad when her offspring leave the nest.   A) ABCDE

B) DEBCA

C) EBDCA

D) AEDBC

Explanation:-   The paragraph talks about the similarities between man and insects but asks us to go beyond the simplistic view about these similarities. The sentence which introduces the comparison first is A. B gives an examples of beetles and wasps to show some characteristics similar to people or humans. C tells how things are done differently by insects vis a vis humans.

Hence the correct sequence is ABCDE.   DIRECTIONS for the question: Read the information given below and answer the question that follows. Question No. : 58 A round table conference was conducted between India and England cricket players. 8 players were present in the conference, of which 4 were British and the rest were Indians. We also know that –   1. Sachin Tendulkar and Anil Kumble were of a short temper and hence had at least one British player nearby. 2. Stuart Broad was feeling like Stuart Little after being hit for six sixes by Yuvraj Singh, so he was not sitting near Yuvraj Singh. 3. Saurav Ganguli never sits between two British players. 4. Sachin Tendulkar was facing north. 5. All the eight players secretly carried different weapons with them in their pockets.Each of the members carried exactly one of the 2 revolvers, 2 pistols, 1 needle, a pair of scissors and 2 knives. 6. Anil Kumble always carried a knife. 7. Stuart Broad never used a pistol. 8. Yuvraj Singh and Kevin Peterson never carried a revolver. 9. The person carrying the needle sat to the left of James Anderson who carried scissors 10. The 2 members who carried revolvers sat together. 11. Sachin Tendulkar never carried a pistol or a revolver and never sat with anyone who carried a pistol or a revolver. 12. Two of the members who faced each other carried a revolver and a pistol respectively. The persons who carried knives faced each other. 13. Paul Collingwood carried a knife.   Name the members who carried revolvers? A) Saurav Ganguli and James Anderson D) Stuart Broad and Saurav Ganguli Explanation:-  

B) James Anderson and Kevin Peterson

C) Saurav Ganguli and Kevin Peterson

  So, Stuart broad and Saurav Ganguli are the person who carried revolver Question No. : 59 Who sat between the members carrying a knife and a pair of scissors? A) Sachin Tendulkar Explanation:-  

B) Yuvraj Singh

C) Kevin Peterson

D) Cannot be determined

  Both Sachin & Kevin Peterson sit between the persons having knife and scissors. So answer can't be determined.

Question No. : 60 What did Kevin Peterson carry? A) Knife

B) Needle

C) Revolver

D) Pistol

Explanation:-  

  As we can see Kevin Peterson carry pistol

Question No. : 61 According to the given conditions, how many seating arrangements are possible? A) 2

B) 3

Explanation:-  

C) 4

D) 1

i.e Only two possible arrangements. Question No. : 62 Who can sit to the right of James Anderson? A) Yuvraj Singh Explanation:-  

B) Kevin Peterson

C) Stuart Broad

D) Paul Collingwood

  In both cases Kevin Peterson sits right to james Anderson.

DIRECTIONS for the question: The question consists of five statements labelled A, B, C, D and E which when logically ordered

form a coherent passage. Choose the option that represents the most logical order. Question No. : 63 A.    A true theory, for example, cannot exist if it is not a reflection on a previous action, having as its goal a return to practice, for it is only in action that theory is confirmed or refuted, developed and enriched. B.    Systematic education as now known -with the school as the privileged point for the diffusion of knowledge, with the professor as the only authorized interpreter of the long accumulated wisdom of the centuries - must disappear. C.    All barriers between “school” and “life” must fall, for they are artificial and false ; they are only one example, among a multitude of others, of the separation imposed by a class society, just like the distinction between manual and intellectual labor, theory and practice, knowledge and action. D.   Against these divisions we must affirm the unity and the dialectical tension in such diverse elements. E.    In effect, life is the real school and education is a permanent process of deciphering what is real, continually redefined and enriched in the light of action and struggle against oppression.   A) BCDEA

B) CBAED

C) BEDCA

D) DABCE

Explanation:-   Option A. Against these divisions – the these here refers to the separation mentioned in C. Hence D will follow C. The passage talks about the systematic education and that it must disappear. Hence the best introductory sentence is B.   DIRECTIONS for the question: Given below are sentences that form a paragraph, identify the sentence(s) or part(s) that is/are incorrect in terms of grammar and usage (including spelling, punctuation and logical consistency). Then, choose the most appropriate option. Question No. : 64 A.      The ultimate objective of development planning is human development or increase social welfare and well-being of the people of a nation. B.      These goal is also important because the sustainability of the development process hinges upon the quality of life enjoyed by the people. C.      A healthy and educated population leads to increased productivity which, in turn, can contribute effectively to output growth. D.      Development strategy, therefore, needs to continuously strive for broad-based improvement in standards of living. E.      High growth is essential to generate resources for social spending.   A) B and D only

B) A and C only

C) A and B only

D) E only

Explanation:-   Option C. In A, we need an adjective to describe social welfare, hence increased instead of increase. In B, This instead of these, as this refers to the ultimate objective of development planning. DIRECTIONS for the question: Identify the most appropriate summary for the paragraph. Question No. : 65 Developing a strong executive leadership culture is not a short term effort. It isn’t based on one person. It almost never deteriorates quickly. Yet markets continually overact to minor blips on the long term success of companies. I think this is mainly due to a failure to appreciate systems and a failure to appreciate variation along with plenty of other contributing factors. The market’s weakness does provide investment opportunities. Though taking advantages of them is much more difficult than

spotting a general weakness. While excellent management almost never becomes pitiful overnight (regardless of how often talking heads would have you believe) business can change very quickly due to rapidly changing market conditions. Avoiding the purchases when the underlying business has sustained a significant blow that excellent management will deal with but which will reduce the value of the enterprise going forward is key to taking advantage of the market’s silly overreaction to bad news (or even calling things “bad news” that are not actually bad just not as awesome as some were hoping for). A) Generally, markets overreact to even minor bad news and if you let this affect your investment decisions, this can have ruinous impact on your market dealings. B)  Being driven by market sentiment when it comes to investment is something that is done by wise investors, who rather focus on the management ability of the company and how that can have a positive impact even in tough business conditions. C)  Sentiment-driven investments, based on market reactions, generally overlook the management strengths of companies and adopt a very myopic vision of the scenario, focusing on the short term rather than the long term. The D) market’s overreaction to bad news is actually an investment opportunity and not a handicap, and if one can focus on the   core strengths of the company, such as skillful management, and understand current business scenarios, there is a definite chance to make significantly correct investment decisions. Explanation:-   Option D In the given case, you need to focus on each and every option closely and then establish the correct answer. Option 4 is the correct answer as it actually highlights the main subject of the paragraph: how there can be an opportunity to invest wisely, even when the market is overreacting to things it should not be. Option 1 is rejected as the central subject is identifying the opportunity to invest and not how one’s market dealings are ruined. Option 2 talks about wise investors, a subject missing from the paragraph (remember the paragraph is about investment opportunities). Option 3 has ‘sentiment driven investments’ as its subject, which is again absent from the passage.  

Question No. : 66

How many boys prefer India and like Hindi movies? A) Can’t say

B) 40

C) 20

D) 50

Explanation:-   The 3 questions and their answers give rise to eight different options and they are as follows:  Ind /Aus

Hin / Eng

Veg / Non veg

Total

Student

I

H

V

35

Chandan

I

H

NV

15

Bhushan

I

E

V

5

 

I

E

NV

45

Eshan

A

H

V

30

Dhiren

A

H

NV

40

Faisal

A

E

V

10

 

A

E

NV

20

Amit

 

 

 

200

 

  From 3, Chandan and 39 others like the same type of food and prefer the same team to win, and of these 34 like the same type of movies. So, the remaining 5 like a different type of movie, which is the triplet India, veg and English. So, Chandan and 34 others like India and veg, but like Hindi. From 5, using the same logic as above, Eshan and 44 others like the same type of movies and teams, as the other 5. Using 7 and 5, we get that Eshan and 44 others like I, E, NV.

From 4, since 35 people like the same type of movies and food as Chandan, and so, Dhiren and 29 others like the combination A, H, V. Similarly, from 2, we can say that Bhushan and 14 others like the combination I, H, NV. Similarly, using the remaining data, we can fill the table as above.   50 boys prefer India and like Hindi movies.

Question No. : 67 Among Amit, Bhushan, Chandan, Dhiren, Eshan and Faisal, how many like vegetarian food? A) 1

B) 2

C) 3

D) 4

Explanation:-   The 3 questions and their answers give rise to eight different options and they are as follows:  Ind /Aus

Hin / Eng

Veg / Non veg

Total

Student

I

H

V

35

Chandan

I

H

NV

15

Bhushan

I

E

V

5

 

I

E

NV

45

Eshan

A

H

V

30

Dhiren

A

H

NV

40

Faisal

A

E

V

10

 

A

E

NV

20

Amit

 

 

 

200

 

  From 3, Chandan and 39 others like the same type of food and prefer the same team to win, and of these 34 like the same type of movies. So, the remaining 5 like a different type of movie, which is the triplet India, veg and English. So, Chandan and 34 others like India and veg, but like Hindi. From 5, using the same logic as above, Eshan and 44 others like the same type of movies and teams, as the other 5. Using 7 and 5, we get that Eshan and 44 others like I, E, NV. From 4, since 35 people like the same type of movies and food as Chandan, and so, Dhiren and 29 others like the combination A, H, V. Similarly, from 2, we can say that Bhushan and 14 others like the combination I, H, NV. Similarly, using the remaining data, we can fill the table as above.   From table above we can see than 2 people - Amit and Dhiren like Veg food. Option 2.   Please note that the total number of Veg lovers is 4, but we are asked preferences of only A, B, C, D, E and F.

Question No. : 68 The total number of boys who preferred Australia winning is A) 12

B) 110

C) 100

D) 90

Explanation:-   The 3 questions and their answers give rise to eight different options and they are as follows: 

Ind /Aus

Hin / Eng

Veg / Non veg

Total

Student

I

H

V

35

Chandan

I

H

NV

15

Bhushan

I

E

V

5

 

I

E

NV

45

Eshan

A

H

V

30

Dhiren

A

H

NV

40

Faisal

A

E

V

10

 

A

E

NV

20

Amit

 

 

 

200

 

  From 3, Chandan and 39 others like the same type of food and prefer the same team to win, and of these 34 like the same type of movies. So, the remaining 5 like a different type of movie, which is the triplet India, veg and English. So, Chandan and 34 others like India and veg, but like Hindi. From 5, using the same logic as above, Eshan and 44 others like the same type of movies and teams, as the other 5. Using 7 and 5, we get that Eshan and 44 others like I, E, NV. From 4, since 35 people like the same type of movies and food as Chandan, and so, Dhiren and 29 others like the combination A, H, V. Similarly, from 2, we can say that Bhushan and 14 others like the combination I, H, NV. Similarly, using the remaining data, we can fill the table as above.   From table above, we can see that total preferring Australia to win is 100.

Question No. : 69 Which of the following is the food, movie, team preference for Dhiren? A) Non-veg, Hindi, Australia

B) Non-veg, English, India

C) Veg, English, India

D) Veg, Hindi, Australia

Explanation:-   The 3 questions and their answers give rise to eight different options and they are as follows:  Ind /Aus

Hin / Eng

Veg / Non veg

Total

Student

I

H

V

35

Chandan

I

H

NV

15

Bhushan

I

E

V

5

 

I

E

NV

45

Eshan

A

H

V

30

Dhiren

A

H

NV

40

Faisal

A

E

V

10

 

A

E

NV

20

Amit

 

 

 

200

 

  From 3, Chandan and 39 others like the same type of food and prefer the same team to win, and of these 34 like the same type of movies.

So, the remaining 5 like a different type of movie, which is the triplet India, veg and English. So, Chandan and 34 others like India and veg, but like Hindi. From 5, using the same logic as above, Eshan and 44 others like the same type of movies and teams, as the other 5. Using 7 and 5, we get that Eshan and 44 others like I, E, NV. From 4, since 35 people like the same type of movies and food as Chandan, and so, Dhiren and 29 others like the combination A, H, V. Similarly, from 2, we can say that Bhushan and 14 others like the combination I, H, NV. Similarly, using the remaining data, we can fill the table as above.   As seen from Table Dhiren speaks Hindi, eats Veg &  prefers win  for Australia.   DIRECTIONS for the question: The question consists of five statements labelled A, B, C, D and E which when logically ordered form a coherent passage. Choose the option that represents the most logical order. Question No. : 70 A.     I haven’t seen a single event — excluding award ceremonies or political events — organised by the power houses where the speakers as well as the audience consist of this section of society. B.    The aam aadmi seems to have only two roles: to make a noise about the issues that hurt him most and provide a foundation for a discourse at a summit by the chosen people; and, to cast a vote that gives the same people an opportunity to continue to be a part of the discourse. C.    Perhaps he or she is not intelligent enough to add to or receive the wisdom that is being purveyed at these events. D.   This way we can conveniently forget the person on whom most of these discourses are based; however if anyone needs to speak and talk about real issues, it is this aam aadmi. E.    I wonder why there cannot be a televised conclave with the same aam aadmi that the politicians, bureaucrats and media houses love to talk about.   A) BDEAC

B) ECDBA

C) DECBA

D) ECBDA

Explanation:-   Option 4. The paragraph talks about the aam aadmi and why he is not included in the discourses which are addressed to him. The he or she in C refers to the aam aadmi. Words like these discourses, this section of society, he or she  tell you that D, B and A cannot be the opening sentence. Hence option 1 and 3 can be ruled out.  D will come after B because the word these in D suggests that there needs to be an earlier sentence which mentions discourse.   DIRECTIONS for the question: Read the passage and answer the question based on it. Question No. : 71 That old joke about the MBA standing for Management By Analysis is no joke at all.   Analysis means "the process of separating something into its con​ stituent elements". Indeed, the word analysis itself comes from a Greek root meaning to unloosen. Sep​ arating things into parts, unloosening them from the whole, is what MBA programs are about. Business becomes a collection of functions; strategy, a set of generic strategies and competitive analyses; even people become analytical things.   The danger of breaking things apart, as Humpty Dumpty discovered to his chagrin, is that it may not be possi​ ble to put them back together. Business schools have not been able to put things back together again because that has to happen in context – in specific situations.          Synthesis is the very essence of management. Within their own con​ texts, managers have to put things together in the form of coherent vi​ sions, unified organizations, integrated systems, and so forth. That is what makes management so difficult, and so

interesting. It's not that managers don't need analysis; rather, it's that they need it as an input to synthesis, and that is the hard part. Teaching analysis devoid of synthe​ sis thus reduces management to a skeleton of itself. This is equivalent to considering the human body as a collection of bones: Nothing holds it together, no sinew or muscle, no flesh or blood, no spirit or soul.   The question remains of where the synthesis comes from. The usual, dismissive, answer is the students: They will put it together. Think of this as the IKEA model of management education: The schools supply the pieces, neatly cut to size; the students do the assembly. Unfortunately, the schools don't supply in​ structions. Worse still, the pieces don't fit together. They may look neat, but in fact they are cut every which way. And the students don't know what to build, because that depends on the situation, and in the class​ room there is no situation, or else several a day in cases. Real management is closer to playing with Lego blocks—there is an infinite number of ways to assemble the pieces, and the interesting structures take time to build.   And so we have this comment from a business school dean, that the prominent consulting firm in which he used to work "officially gave up on the notion that graduates of the Harvard Business School or Stanford knew anything more about thinking integrative about business prob​ lems than a Swarthmore or Amherst or Williams undergraduate in phi​ losophy".   Excerpted from ‘Managers Not MBAs’ by Henry Mintzberg   Which of the lines, quoted from the passage, is the main message that the author is trying to convey through this passage? A) Games like Lego help harness the creative talents of management B) Teaching analysis devoid of synthe​ sis thus reduces management to a skeleton of itself. C) Analysis is the very essence of management. D) Business is a collection of functions. Explanation:-   Option B. The author is stating that while management schools all teach analysis - breaking things into parts, in reality management involves synthesis - managers have to put things together in the form of coherent vi​ s ions, unified organizations, integrated systems, and so forth. Hence option 2 is the best option.

  1 - The Lego game example is used to only draw an analogy   3 - It is clearly stated that Management by Analysis is not the best thing to do   4 - He clearly states the reverse when he mentions that this is akin to treating the human body as a collection of bones   DIRECTIONS for the question: Read the passage and answer the question based on it. Question No. : 72 What change does the author seem to be suggesting to deans of MBA Schools?

A) Instead of talking, analyzing and deciding, students need to work on the doing, seeing, feeling and listening. B) Instead of taking in students who are fresh from college or with very less experience, concentrate on those whose work experience includes supervisory skills. C) Instead of focusing exclusively on application, spend at least half the time making students learn basic subjects like arts and literature. D) A higher emphasis on the practice, rather than the theory of management. Explanation:-   Option A. According to the author " Business schools have not been able to put things back together again because that has to happen in context – in specific situation." The author also thinks that integrative thinking about business problems which would involve doing, seeing, feeling and listening is not being encouraged in business schools. Hence option 1 is the best option.

 

2 - Most US B schools are already doing this.

 

3 - Not mentioned in the passage. Also it Is Ok for an undergraduate program, but not so for an MBA program

 

4 - Management is all practice in any case, so this option is redundant.

DIRECTIONS for the question: Read the passage and answer the question based on it. Question No. : 73 All of the following can be concluded/inferred from the passage except: A) Business analysis without the accompaniment of the suitable gumption of synthesis is like a sack of bones, with all the parts there but the connecting elements missing. B)Swarthmore or Amherst or Williams undergraduates in philosophy know more about management than the ones at   Harvard Business School or Stanford. C) Analysis, as referred to in the passage, risks a possible outcome where it might not be possible to construct a holistic solution from the de-constructed parts achieved by the analytic process in the first step. D)  Remarkable solutions in management take time to build as one needs to identify the best approach from a plethora of different options that enable one to put in order the different parts of the puzzle one is faced with. Explanation:-   Correct answer: Option B. Option A can be inferred/concluded from the lines: Teaching analysis devoid of synthe​ sis thus reduces management to a skeleton of itself. This is equivalent to considering the human body as a collection of bones: Nothing holds it together, no sinew or muscle, no flesh or blood, no spirit or soul.   Option B is the incorrect inference. Refer to the lines: And so we have this comment from a business school dean, that the prominent consulting firm in which he used to work "officially gave up on the notion that graduates of the Harvard Business School or Stanford knew anything more about thinking integrative about business prob​ lems than a Swarthmore or Amherst or Williams undergraduate in philosophy". The passage states that a Harvard Business School or Stanford knew anything more about thinking integrative about business problems than a Swarthmore or Amherst or Williams undergraduate in philosophy, in other words, they might be at the same level. It does not mean that Swarthmore or Amherst or Williams undergraduates in philosophy know more about management that ones at Harvard Business School or Stanford.   Option C can be inferred/concluded from the lines: The danger of breaking things apart, as Humpty Dumpty discovered to his chagrin, is that it may not be possi​ ble to put them back together.   Option D can be inferred/concluded from the lines: Real management is closer to playing with Lego blocks—there is an infinite number of ways to assemble the pieces, and the interesting structures take time to build. DIRECTIONS for the question: Read the passage and answer the question based on it. Question No. : 74 The main hallmark of the IKEA model of management education is A) An approach which ensures a mix of real life experience with in class theory. B) A systems approach, with all components defined and ready for assembly. C) A type of management akin to coaching, that which also emphasizes independence.

D) If it ain’t broke, don’t fix it.

Explanation:-   Option B. The passage states "Think of this as the IKEA model of management education: The schools supply the pieces, neatly cut to size; the students do the assembly. Unfortunately, the schools don't supply in​ s tructions. Worse still, the pieces don't fit together." the word unfortunately tells us that in the original IKEA model, the instructions are stated and the pieces fit perfectly. Hence option 2 is the best answer.

 

Option 1 is incorrect because according to the author real life is like lego bricks and not IKEA.

 

Option 3 is incorrect because there is no independence in IKEA - the shapes, sizes are all fixed.

 

Option 4 is incorrect because IKEA talks of building new and not of reparing anything old.

DIRECTIONS for the question: Read the passage and answer the question based on it. Question No. : 75 One of the key criticisms which is in tune with the philosophy espoused in the passage is reflected in which of the statements below? A) Business schools are never able to get the balance right when they decide the percentages of the two approaches of management, that is analysis and synthesis. B) MBA schools are lop-sided in their evaluation of candidates because of an over-reliance on analytical testing tools. C) MBA programs lack the precise and defining features that would enable their takers to solve real world problems; instead they harp on management jargon and platitudes. D)MBA programs commit the mistake of missing out on crucial instructions and approaches that would enable their takers to   offer integrated solutions to problems. Explanation:-   Option D. The context of the passage is all about synthesis and the lack of it in management programs. This helps us rule out option A, as the passage at no point talks about there being a tug of war between the two approaches. Option B is an unrelated option as it talks about selection procedure rather than what is being done in MBA schools. Option C is vague in nature and does not spell out the actual problem. Option D is the precise answer and its context can be found in the lines: And so we have this comment from a business school dean, that the prominent consulting firm in which he used to work "officially gave up on the notion that graduates of the Harvard Business School or Stanford knew anything more about thinking integrative about business prob​ lems than a Swarthmore or Amherst or Williams undergraduate in phi​ losophy".

DIRECTIONS for the question: Identify the most appropriate summary for the paragraph. Question No. : 76 There is nothing new about companies adapting their products to the pockets and preferences of emerging-market consumers. Unilever and Procter & Gamble started selling shampoo and washing powder in small sachets more than two decades ago to cater for customers with cramped living spaces and even more cramped budgets. Nike produces an all-enveloping athletic uniform to protect the modesty of Muslim women athletes. Mercedes puts air-conditioning controls in the back as well as the front of its cars because people who can afford a Mercedes can also afford a driver. But GE and TCS are doing something more exciting than fiddling with existing products: they are taking the needs of poor consumers as a starting point and working backwards. Instead of adding ever more bells and whistles, they strip the products down to their bare essentials. A) The era of comparative innovation here, with companies striving to outdo products of the previous generations by virtue of innovation and frugality. B) The era of minimalistic products, which serve the essential needs of the maximum possible, is here. C) The era of basic products, serving the maximum possible number of people and providing affordable options, is here with the help of companies will to add new dimensions to product creation.  D)The era of inventive minimalist products, which serve the purpose of those in need, is here with the help of companies   willing to adapt products with respect to their target audiences. Explanation:-   Option D Option 1 introduces a new subject, ‘comparative innovation’, which finds no mention in the passage. Option 3 is ruled out on two counts: the passage does not mention that products are meant for the maximum number of people and secondly instead of adding, companies are stripping down their products. Option 2 is ruled out on similar grounds to option 3. Option 4 is the perfect option as it includes all the aspects of the paragraph and does not use wording that implies any extreme scenario.  

DIRECTIONS for the question: Read the passage and answer the question based on it. Question No. : 77 Imagine living in a country where having the freedom to cultivate your own land, tax-free and without government interference, is not only common but also encouraged for the purpose of promoting individual sovereignty and strong, healthy communities. Now imagine that in this same country, nearly all of your neighbors also cultivate their own land as part of a vast network of decentralized, self-sustaining, independent “eco-villages” that produce more than enough food to feed the entire country.   You might be thinking this sounds like some kind of utopian interpretation of historical America, but the country actually being described here is modern-day Russia. It turns out that Russia’s current agricultural model is one that thrives as a result of the millions of small-scale, family-owned and -operated, organically-cultivated farms that together produce the vast majority of the food consumed throughout the country.   A far cry from the unsustainable, chemical-dependent, industrialized agriculture system that dominates the American landscape today, Russia’s agricultural system, which is not technically a system at all, is run by the people and for the people. Thanks to government policies there that actually encourage autonomous family farming, rather than cater to the greed of chemical and biotechnology companies like they do here in the States, the vast majority of Russians are able and willing to grow their own food on privately-owned family plots known as “dachas.”   According to The Bovine, Russia’s Private Garden Plot Act, which was signed into law back in 2003, entitles every Russian citizen to a private plot of land, free of charge, ranging in size from 2.2 acres to 6.8 acres. Each plot can be used for growing food, or for simply vacationing or relaxing, and the government has agreed not to tax this land. And the result of this effort has been phenomenal, as Russian families collectively grow practically all the food they need.   “Bear in mind that Russia only has 110 days of growing season per year — so in the U.S., for example, gardeners’ output could be substantially greater. Today; however, the area taken up by lawns in the U.S. is two times greater than that of Russia’s gardens — and it produces nothing but a multi-billion-dollar lawn care industry.”   Excerpted from the journal ‘The Ringing Cedars’ edited by Leonid Sharaskin   Which of the following represents an appropriate title for this passage? A) A simple fix for farming that the modern day world cannot live without.  B) Russia’s small scale organic agriculture model may hold the key to feeding the world. C) Russian Farmers have a fruitful year and they hope to keep their barns flowing with fresh produce. D) Getting more bang from your lawn buck: simultaneously revolutionize the way you farm and mow your lawn. Explanation:-   Option B. The passage states that Russian way of agriculture is keeping it food-sufficient. Families are working together to make themselves food sufficient.  The passage also says that this is not utopian which means we can put it into practice.  Hence the ‘Russia way’ may be the way to make the world sufficient. This makes option B the best answer. Option A, though a generic answer, cannot be correct as it is too strong as answer option and represents a viewpoint that may not be always in play. Options C and D are too vague in the given context and do not fit the sentiment of the passage. DIRECTIONS for the question: Read the passage and answer the question based on it. Question No. : 78 From the following words/phrases, identify the ones that can be used to define Russia’s agricultural system: I. Laissez-faire II. Institutionalized III. Quixotic IV. Free-for-all V. Self-reliant

A) I, III & IV

B) I & V

C) III, IV & V

D) II & V

Explanation:-   Option B Analyzing the words/phrases one at a time: I. Laissez-faire: Laissez-faire in a doctrine which has its precepts in individualism. It is one that states that the government should not interfere in commercial affairs and places no restrictions on trade. In the given context, it perfectly fits the method of operations of Russian Agriculture. II. Institutionalized: This option is diametrically opposite to what is stated in the passage. The Russian system is de-centralized, not institutionalized. III. Quixotic: Quixotic implies being romantic in an unrealistic and dreamy way. It has sentiments of foolishness attached to it and that is certainly not the case here. IV. Free-for-all: Contrary to popular interpretations, free-for-all means ‘a disorderly fight, argument, or competition in which everyone present participates’ and finds no relevance in the given context of the passage. V. Self-reliant: This word is the perfect description for the Russian agricultural system.

DIRECTIONS for the question: Read the passage and answer the question based on it. Question No. : 79 According to the passage, Russia’s agricultural model and that of America’s are: A) mutually antagonistic

B) irreconcilably different

C) fundamentally dissimilar

D) theoretically analogous

Explanation:-   Option C. According to the information given in the passage, there is one safe conclusion that we can draw about the two agricultural models: they are completely different to each other and there is no common ground between them. Let us analyze the options against this backdrop: Option A pits them as enemies, a relationship that cannot exist between two models. Option B uses the word ‘irreconcilably’, meaning that something cannot be made compatible with another and no agreement can be reached between the two. As with option A, we are not trying to achieve any sort of reconciliation in the given case. Option C is the most simple and clear-cut phrasing in this case. Option D completely goes against the sentiment of the passage. DIRECTIONS for the question: Given below are sentences that form a paragraph, identify the sentence(s) or part(s) that is/are incorrect in terms of grammar and usage (including spelling, punctuation and logical consistency). Then, choose the most appropriate option. Question No. : 80 A.    In view of the fact that the significance of a commitment to lifelong learning and B.    lifetime education are being discussed only in recent years, even in educationally advanced countries, C.    the possibility of the idea becoming an integral part of educational thinking seems to be a far cry. D.    For, to move in that direction means much more than some simple rearrangement of the present organisation of education. E.    But a good beginning can be made by developing open university programmes of different categories for elder learners and introducing extension services in the conventional colleges and schools.   A) B and D only

B) B and E only

C) E only

D) A and C only

Explanation:-   Option 2. In B, “is being “ instead of “are being”, as the subject is the significance. In E, older and not elder. We usually use elder if we are talking of the same family.

DIRECTIONS for the question: Read the information given below and answer the question that follows. Question No. : 81 7 cousins, Mandar, Vivek, Charu, Nilima, Santosh, Lourdes and Shreya have their birthdays on the same day in the same month, but are all born in different years. Santosh is the eldest at 24 years while Lourdes is the youngest at 18 years. Charu’s age is between the ages of Shreya and Nilima and Vivek is older than Charu. What could be Charu’s age? A) 20 or 21

B) 21 or 22

C) 19 or 20

D) 19 or 21

Explanation:-  

Either Shreya or Nilima is elder to Charu. Also, Vivek is elder to Charu. If Mandar is younger than Charu, Charu will be the 4th eldest and her age will be 21. However, if Mandar is elder to Charu, then Charu will be the 5th eldest and her age will be 20. Question No. : 82 Four political parties, A, B, C and D contested an election by nominating one candidate each to each of the assembly seats. Party A lost in 28 of the seats; party B lost in 21 of the seats; party C lost in 25 of the seats; party D lost in 31 of the seats. What was the total number of assembly seats? A) 30

B) 35

C) 45

D) 49

Explanation:-   Work with the options to get the answer as 35.Alternately, Suppose party D wins x seats. The total number of seats is 31 + x. Parties A, B and C will win (x + 3), (x + 10) and (x + 6) seats respectively. So, the total number of seats won by all 4 parties is (4x + 19) and the total number of seats lost by them is 105. Since a seat can be won by 1 party and lost by 3 parties, 3 × (4x + 19) = 105 ⇒ x = 4. Thus the total number of seats is 31 + 4 = 35. Question No. : 83 At a management course, four students of the first year, Anamika, Bhakti, Chetan and Dinesh team up with four students of the second year, Esha, Fatima, Geeta and Himani. Chetan and Dinesh are males while the others are females. They form groups with distinct number of students so that each group has exactly one male student. Chetan is part of a group that has neither Geeta nor Fatima. Bhakti and Geeta are part of the same group. Esha is in a group that does not have Dinesh. Which of the following is always true? A) Chetan is in a group with Anamika and Esha B) Dinesh is in a group with Bhakti and Geeta C) Chetan is in a group with Anamika D) Himani is in a group with Fatima and Bhakti Explanation:-   Chetan and Dinesh must be in 2 different groups. Since the number of students in each of the groups is distinct, we can form 2 groups of 3 and 5 students or 2 groups of 2 and 6 students. From the given information, we can conclude that Bhakti, Geeta and Fatima are in the same group as Dinesh and Esha is in the same group as Chetan. If Chetan and Esha are the only 2 members in their group, then the other group of 6 will be Dinesh, Bhakti, Geeta, Fatima, Anamika and Himani.On the other hand, 2 groups of 3 and 5 can be formed in 2 ways.The group of 3 will comprise Chetan, Esha and Anamika while the group of 5 will comprise Dinesh, Bhakti, Geeta, Fatima and Himani. Or, the group of 3 will comprise Chetan, Esha and Himani while the group of 5 will comprise Dinesh, Bhakti, Geeta, Fatima and Anamika. In all cases, Dinesh is in the group with Bhakti and Geeta. Question No. : 84 6 candidates, Apurva, Kaushik, Minoti, Subahu, Tejas and Yash, are scheduled to appear for their interviews at a Business School. The candidates were interviewed one at a time so that Apurva was interviewed after Kaushik. Minoti was interviewed either immediately before or immediately after Yash. Subahu was interviewed neither immediately before nor immediately after Yash. If Tejas was the fourth candidate to be interviewed, which of the following cannot be the order, from first to sixth, in which the candidates were interviewed? A) Subahu, Minoti, Yash, Tejas, Kaushik, Apurva C) Minoti, Yash, Kaushik, Tejas, Apurva, Subahu

B) Kaushik, Yash, Minoti, Tejas, Subahu, Apurva D) Kaushik, Subahu, Yash, Tejas, Minoti, Apurva

Explanation:-   According to the data, Subahu cannot be interviewed immediately before or immediately after Yash. Option 4 violates this condition and must be the answer. DIRECTIONS for the question: Answer the question based on the information given in the passage. Question No. : 85 The coming-in of private airlines in the aviation sector has actually had one unintended consequence: the delays at the top 5 airports in the country have increased by 40%. The aviation ministry took up a project to assess the reasons for this delay and came out with the proposal of constructing a new terminal and runaway at each of these airports.    The argument of the aviation ministry is based on all of the following assumptions except: A) The existing airports have the additional land available for considerable development to take place. B) From the time the private airlines began operation, the number of airplanes in the country has increased. C) The existing passenger load at the top 5 airports necessitates an increase in increase in the number of slots available for flights.  D) Existing airports in the top 5 cities do not have the required number of slots that can be allocated to all airlines.  Explanation:-   Option B In this case, you need to identify 3 assumptions made by the aviation ministry. One of the options will not be the assumption. In this case, that option is option 2. Remember the conclusion of the aviation ministry:  a new terminal and runaway are needed at each of these airports, that is capacity expansion needs to be carried out. How can such a recommendation be made? It depends on three assumptions: 1. Sufficient availability of land for expansion 2. Sufficient consumers available for expanded capacity. 3. Existing infrastructure not enough to accommodate planes (implied by option 4). The number of planes in the country does not affect the given conclusion.   Question No. : 86 The coming-in of private airlines in the aviation sector has actually had one unintended consequence: the delays at the top 5 airports in the country have increased by 40%. The aviation ministry took up a project to assess the reasons for this delay and came out with the proposal of constructing a new terminal and runaway at each of these airports.    Some of the following statements weaken the argument of the aviation ministry:   I. The most important cause of delay is lack of fog detection equipment at these airports, which are based in cities all impacted by fog. II. The air traffic control equipment at these airports has not been changed for the last two decades and it frequently malfunctions. III. Commercial flights at these airports account for 20% of the flight traffic. IV. Private airlines, in order to poach customers, are running competing flights on same time, even thought occupancy for these flights is less than 30% each.   Identify the number of above statements that weaken the argument of the aviation ministry except:   A) I

B) II

C) III

D) IV

Explanation:-   Option C In the given case, three statements weaken the argument of the airline industry. In order to prove that the aviation ministry’s argument is flawed, we need to show that the addition of a new runway and terminal would not solve the given problem. Let us analyze each statement and see how it works in the given context:

Statement I: It provides another reason for delay and hence weakens the aviation ministry’s argument. Statement II: It provides another reason for delay and hence weakens the aviation ministry’s argument. Statement III: In fact, it strengthens the argument by showcasing that 20% of the airport’s capacity is indulged in commercial activities. Statement IV: It provides a scenario wherein capacity expansion at the airport will not help. The problem is the excess number of flights being run and this means adding another runaway might complicate matters further. Thus, three statements weaken the aviation ministry’s argument.   DIRECTIONS for question  The question consists of different statements  one of which does not fit into the paragraph. Identify that sentence which is a misfit and choose that option as your answer. Question No. : 87 A. In addition to having a general historical background, historians of technology education need to become familiar with specialized bodies of historical literature such as the history of technology, social history, or history of education. B. The issues central to this essay arise out of recent literature in the history of technology, especially as reflected in the work of members of the Society for the History of Technology (SHOT) in its journal Technology and Culture. C. Within SHOT, there has been much reflection on technology, its historical relationships to other aspects of society, and alternative approaches to writing history. D. But as recently as 1974, two authors at a SHOT meeting, stressed the lack of a "conceptual framework" for the history of technology. A) Statement A

B) Statement B

C) Statement C

D) Statement D

Explanation:-   Statement A is pretty much generic as it talks of what is required of historians of technology education. The rest refer to SHOT and its proceedings and form a proper sequence B-C-D.

DIRECTIONS for the question: The question consists of five statements labelled A, B, C, D and E which when logically ordered form a coherent passage. Choose the option that represents the most logical order. Question No. : 88 A. The function of the world government is to prevent war, and it should have only such powers as are necessary to this end. B. The general principle should be to leave to smaller bodies all functions which do not prevent the larger bodies from fulfilling their purpose. C. This involves a monopoly of armed force, a power to sanction and revise treaties, and the right to give decisions in disputes between States. D. The problem of delimiting the powers of various bodies will be one presenting many difficulties. E. Confining ourselves, for the moment, to geographical bodies, there should be a hierarchy from the world governments to parish councils.   A) DCABE

B) BCDEA

C) ECDBA

D) DBEAC

Explanation:-   Option D The passage talks about delimiting powers and hence D is the best opening sentence. What the delimiting should involve is spoken of in B. In order to prevent was what is required is mentioned in C. Hence C will follow A. Hence the sequence is DBEAC. DIRECTIONS for question  The question consists of different statements  one of which does not fit into the paragraph. Identify that sentence which is a misfit and choose that option as your answer. Question No. : 89

A. Innocence insisted on too strenuously is tantamount to guilt. B. But it's precisely the problem; the fact that human evil is predictable does not make it excusable. C. It's human nature to want to believe in the rightness of our own actions and intentions. D. Nor does fobbing off the evil on singularly evil individuals like Hitler or bin Laden or Cheney. A) Statement A

B) Statement B

C) Statement C

D) Statement D

Explanation:-   Option A   The rest three option are connected together and form a trio.The order will be CBD. Statement A does not fit anywhere. Question No. : 90 A. Smart businessmen aren't interested in putting their efforts into business plans dependant on laws which, not only haven't been passed yet, but haven't even been introduced. B. Smart Congressmen aren't interested in putting their efforts into introducing legislation to create industries that businessmen aren't even talking about yet. C. It would make a huge difference if someone capable of operating at that level, such as a really big investor or a Fortune 500 company, took the time to think carefully about the economics and realized how profitable owning land on the Moon could be and talked of the necessary legal eco-system. D. Of course, the Congress would give the idea much more credibility if they heard someone representing Boeing or Lockheed tell them that passing a land claims recognition law would lead to a serious privately funded space development effort but the problem, unfortunately, has been like the classic "which comes first: the chicken or the egg?" A) Statement A

B) Statement B

C) Statement C

D) Statement D

Explanation:-   Option A The lines B, C and D given here are interlinked as they all talk of the idea of the Congress enabling suitable laws to make possible the ownership of land in the outer space. Contrastingly, statement A talks of businessmen’s reluctance to venture into something unless it has the enabling legal provisions. DIRECTIONS for the question : Read the passage and answer the question based on it.  Question No. : 91 It is not always a profound interest in man that carries travellers nowadays to distant lands. More often it is the facility for rapid movement. For lack of time and for the sake of convenience we generalise and crush our human facts into the packages within the steel trunks that hold our travellers' reports.   Our knowledge of our own countrymen and our feelings about them have slowly and unconsciously grown out of innumerable facts which are full of contradictions and subject to incessant change. They have the elusive mystery and fluidity of life. We cannot define to ourselves what we are as a whole, because we know too much; because our knowledge is more than knowledge. It is an immediate consciousness of personality, any evaluation of which carries some emotion, joy or sorrow, shame or exaltation. But in a foreign land we try to find our compensation for the meagreness of our data by the compactness of the generalisation which our imperfect sympathy itself helps us to form. When a stranger from the West travels in the Eastern world he takes the facts that displease him and readily makes use of them for his rigid conclusions, fixed upon the unchallengeable authority of his personal experience. It is like a man who has his own boat for crossing his village stream, but, on being compelled to wade across some strange watercourse, draws angry comparisons as he goes from every patch of mud and every pebble which his feet encounter.   Our mind has faculties which are universal, but its habits are insular. There are men who become impatient and angry at the least discomfort when their habits are incommoded. In their idea of the next world they probably conjure up the ghosts of their slippers and dressing-gowns, and expect the latchkey that opens their lodging-house door on earth to fit their front door in the other world. As travellers they are a failure; for they have grown too accustomed to their mental easy-chairs, and in their intellectual nature love home comforts, which are of local make, more than the realities of life, which, like earth itself, are full of ups and downs, yet are one in their rounded completeness.  

The modern age has brought the geography of the earth near to us, but made it difficult for us to come into touch with man. We go to strange lands and observe; we do not live there. We hardly meet men: but only specimens of knowledge. We are in haste to seek for general types and overlook individuals. When we fall into the habit of neglecting to use the understanding that comes of sympathy in our travels, our knowledge of foreign people grows insensitive, and therefore easily becomes both unjust and cruel in its character, and also selfish and contemptuous in its application. Such has, too often, been the case with regard to the meeting of Western people in our days with others for whom they do not recognise any obligation of kinship.   It has been admitted that the dealings between different races of men are not merely between individuals; that our mutual understanding is either aided, or else obstructed, by the general emanations forming the social atmosphere. These emanations are our collective ideas and collective feelings, generated according to special historical circumstances.   In the context of the passage, the author is primarily concerned with: A) exposing the archetypical biases of the Western man about others in the world.  B) highlighting the thoughts of men when they travel, and how these might be insufficient to describe the landscape they travel to. C) proving that most men are not good travelers and suffer from biases that help them establish a blinkered vision of the place they are travelling. D) pointing out that all men may not observe and communicate the unbiased appraisals of the places they travel as they might   be affected by their our prejudicial understanding of the world.  Explanation:-   Option D  Refer to the lines "It is an immediate consciousness of personality, any evaluation of which carries some emotion, joy or sorrow, shame or exaltation. But in a foreign land we try to find our compensation for the meagreness of our data by the compactness of the generalisation which our imperfect sympathy itself helps us to form. When a stranger from the West travels in the Eastern world he takes the facts that displease him and readily makes use of them for his rigid conclusions, fixed upon the unchallengeable authority of his personal experience. It is like a man who has his own boat for crossing his village stream, but, on being compelled to wade across some strange watercourse, draws angry comparisons as he goes from every patch of mud and every pebble which his feet encounter."   Thus option D is the prime concern of the author.     DIRECTIONS for the question : Read the passage and answer the question based on it.  Question No. : 92 The example of the village stream is used by the author to prove:   I. In comparison to the world, how insular Western men are. II. How insular habits of the mind can lead us to form biased conclusions. III. How men, when placed in inconvenient circumstances, can form rigid conclusions on the basis of that limited evidence. (Select the statements that answer the question above)   A) I & II

B) II & III

C) I & III

D) All of the above

Explanation:-   Option B Statement I cannot be derived as no comparison is implied by the said example. Statement II can be derived from the lines: Our mind has faculties which are universal, but its habits are insular. Statement III can be derived from the lines: When a stranger from the West travels in the Eastern world he takes the facts that displease him and readily makes use of them for his rigid conclusions, fixed upon the unchallengeable authority of his personal experience.  

DIRECTIONS for the question : Read the passage and answer the question based on it.  Question No. : 93 It can be inferred from the passage that: A) Travelling has been made easy in the modern day world with the help of innovations that the world has seen.  B) Western people do not generally recognize any obligation of kinship when they travel. C)Travelling can be fruitful to form opinions of the world when the traveler dissolves himself in his reference frame rather   than simply commenting on it. D) Both options 1 and 3 Explanation:-   Option 3 can be inferred from the lines: The modern age has brought the geography of the earth near to us, but made it difficult for us to come into touch with man. We go to strange lands and observe; we do not live there. We hardly meet men: but only specimens of knowledge….When we fall into the habit of neglecting to use the understanding that comes of sympathy in our travels….   Option 1 is simply a fact that is being repeated and is not an inference in the given case. Option 2 is a partial fact picked from the passage that does not qualify as an inference.    DIRECTIONS for the question : Read the passage and answer the question based on it.  Question No. : 94 It can be derived from the passage that a person places focus on which of the two qualities in his own land and in foreign land (in that order): A) mutability and inadequacy D) liberalism and parochialism

B) fluidity and parochialism

C) exaltation and meagerness

Explanation:-   Option B Refer to the lines: They have the elusive mystery and fluidity of life. We cannot define to ourselves what we are as a whole, because we know too much; because our knowledge is more than knowledge. It is an immediate consciousness of personality, any evaluation of which carries some emotion, joy or sorrow, shame or exaltation. But in a foreign land we try to find our compensation for the meagreness of our data by the compactness of the generalisation which our imperfect sympathy itself helps us to form. The portions in bold above helps us identify the correct answer in this case. Fluidity is directly given in the passage and parochialism can be derived. Compactness of the generalisation can be labeled as parochialism (a limitation of views or interests like that defined by a local parish).   DIRECTIONS for the question: Given below are sentences that form a paragraph, identify the sentence(s) or part(s) that is/are incorrect in terms of grammar and usage (including spelling, punctuation and logical consistency). Then, choose the most appropriate option. Question No. : 95 A.  What an idea it had been that had sent me so audaciously trespassing that I could not remember . B.   The spirit of peace dissented like a cloud from heaven, for if the spirit of peace dwells anywhere,   C.   it is in the courts and quadrangles of Oxbridge on a fine October morning. D.  While strolling through those colleges past those ancient halls the roughness of the present seemed smoothed away; E.   the body seems contained in a miraculous glass cabinet through which no sound could penetrate.  

A) A and E only

B) B only

C) B, D and E

D) A, B and D

Explanation:-   Option C In B, it should be descended (moving downwards) and not dissented (disagree). In D, it should be strolling and not while strolling. In E, it should be seemed to be and not seems.

DIRECTIONS for the question : Read the passage and answer the question based on it.  Question No. : 96 Anyone with leadership ambitions in the BSP runs smack into the obvious lack of opportunity to represent the party in either the state assembly or Parliament. The overwhelming majority of the election tickets in polls are distributed on a piecemeal basis to candidates, depending on their caste, community and the financial resources at their command. They do not even have to be BSP members.   A distinctive trait of the BSP has been its reluctance to be involved in mass agitation that would pit the party cadre against the state, instead emphasizing the priority of political power over social reforms. The reason is in the ideological line adopted by Kanshi Ram. The BSP founder had single mindedly focused on the pursuit of Ambedkars’s ‘master key of political power’ which he felt was essential to have before he could change society. The cadre has instead devoted all its organizational energies towards promotional activities projecting the party and its leaders mostly in preparation for the next elections. Neither Kanshi Ram nor Mayawati belonged to the political mainstream. There was no compulsion to observe the political conventions and codes which bound most parties. The two leaders regarded everyone who was involved in the growth of BSP as functional instruments who were dispensable. This also explains why they rejected a functional multi-tiered hierarchial party organization.   How has the BSP managed to retain the loyalty of oppressed groups without agitating on their behalf? Mayawati has supplemented a hollow shell of a party with a formidable election machine that runs on its own steam. The main motivation for the hundreds of thousands who have joined the BSP is the extra clout they acquire in their social circle by doing so. This is especially true among Dalits who still form the bulk of party cadre, but also amongst poorer backward castes and Muslims. The reflected glory they get from being associated with Mayawati and her party helps in many tangible ways. In the beginning this helped them enhance their stature mainly among their families and within their ethnic group. But as Behenji grows from strength to strength, earning the fear and respect of all castes and communities across UP, joining the BSP increased the party member’s standing in society at large.   Having a marginalized sub-altern group like the Dalits as her core constituency, she has been able to get away with the authoritarian methods that may have been unacceptable to more socially secure communities. It is ironic that the wretched of the earth have been historically prone to hero worship a dominant figure who brooks no dissent. Kanshi Ram had deliberately cast his protégé in the mould of such a goddess, cleverly packaged in the more familiar reassuring form of ‘Behenji’ (respected sister)   Excerpted from ‘Behenji: A political biography of Mayawati’ by Ajoy Bose. Why has the BSP been not participating in mass agitations? A) The BSP believes in emphasizing on political power rather than on social reforms B) The BSP believes in not supporting any social cause C) It believes that mass agitations do improve the standing of the party albeit vicariously D) It believes that small, yet focussed mutiny give better results than mass revolts Explanation:-   Option A Option 4 is incorrect as it does believe in revolts at all. It believes that all its organizational energies should be devoted towards promotional activities projecting the party and its leaders mostly in preparation for the next elections. The issues of the society have no place in their working. It is nowhere directly mentioned that they do not believe in supporting any social cause. Vicarious or otherwise it does not believe in mass agitation. DIRECTIONS for the question : Read the passage and answer the question based on it. 

Question No. : 97 How has Mayawati used her government in changing society? A) She had improved and enhanced the standing of the Dalits and other marginalized groups in society B) She has used her power to bring more reservations in the central government jobs to the marginalised groups C) She had improved the standard of living of all her party members D) Her dictatorial functioning has made the central government bend their rules in case of Uttar Pradesh Explanation:-   Option A There is no mention of her being able to twist the arm of the government at the centre. Hence option 4 is incorrect. She has improved the status but if that means improving every party member’s economic condition is debatable. Hence option 3 is incorrect. Reservations is not mentioned in the passage. DIRECTIONS for the question : Read the passage and answer the question based on it.  Question No. : 98 What is implied as a disadvantage of the direct channel used by Mayawati to control her party functionaries? A) There is no proper distribution of jobs in the party B) No noticeable hierarchy exists C) Hierarchy is a hypocrisy in the party and yet others follow blindly D) There is no second line of leadership, so succession is going to be an issue Explanation:-   Option D Power is concentrated in the hands of the Mayawati and others do not have any role to play. This will lead to a succession problem and could lead to the wiping out of the party or total chaos after Mayawati disappears from the picture. Option 2 and 3 though true do not bring out the disadvantage as explicitly as option 4. DIRECTIONS for the question : Read the passage and answer the question based on it.  Question No. : 99 Why does the author think that socially secure communities do not like dictators? A) They are not as prone to hero worship as the Dalits are B) They can stand on their own and are not dependent on anyone C) These communities believe in being only leaders and not followers D) They are cocooned in their sense of security and appreciate being led with an iron hand Explanation:-   Option A Dalits would look up to anyone who fights for their cause as they have been the one who have been handed a raw deal. Option 3 is incorrect.Secure communities also have leaders, though they may not look up to dictators. Option 4 is incorrect and they do not like to be dictated to. DIRECTIONS for question  The question consists of different statements  one of which does not fit into the paragraph. Identify that sentence which is a misfit and choose that option as your answer. Question No. : 100 A) It is clear that the original making of random sounds, without which speech would never be learnt, is instinctive B) This applies with great force to man, so much so that some have thought instinct less important in the life of man than in the life of animals

life of animals C) What is true of animals, as regards instinct and habit, is equally true of men D) But the higher we rise in the evolutionary scale, broadly speaking, the greater becomes the power of learning, and the fewer are the occasions when pure instinct is exhibited unmodified in adult life Explanation:-   Option A The paragraph talks about instinct with reference to man and animal. The sequence is 3-4-2 , leaving 1 as the odd one out. Option 1 probably follows a few sentences after as it also talks about instinct but with reference to sounds and speech.

View more...

Comments

Copyright ©2017 KUPDF Inc.
SUPPORT KUPDF